Difference between revisions of "2020 AMC 10A Problems/Problem 12"

(Solution where the figure is drawn out and area is approximated deleted due to inability to directly construct figure with given information)
(Undo revision 199453 by Hijacky (talk))
(Tag: Undo)
(65 intermediate revisions by 24 users not shown)
Line 1: Line 1:
== Problem ==
+
== Problem 12 ==  
 +
Triangle <math>AMC</math> is isosceles with <math>AM = AC</math>. Medians <math>\overline{MV}</math> and <math>\overline{CU}</math> are perpendicular to each other, and <math>MV=CU=12</math>. What is the area of <math>\triangle AMC?</math>
  
Triangle <math>AMC</math> is isoceles with <math>AM = AC</math>. Medians <math>\overline{MV}</math> and <math>\overline{CU}</math> are perpendicular to each other, and <math>MV=CU=12</math>. What is the area of <math>\triangle AMC?</math>
+
<asy>
 +
draw((-4,0)--(4,0)--(0,12)--cycle);
 +
draw((-2,6)--(4,0));
 +
draw((2,6)--(-4,0));
 +
label("M", (-4,0), W);
 +
label("C", (4,0), E);
 +
label("A", (0, 12), N);
 +
label("V", (2, 6), NE);
 +
label("U", (-2, 6), NW);
 +
label("P", (0, 3.6), S);
 +
</asy>
  
 
<math>\textbf{(A) } 48 \qquad \textbf{(B) } 72 \qquad \textbf{(C) } 96 \qquad \textbf{(D) } 144 \qquad \textbf{(E) } 192</math>
 
<math>\textbf{(A) } 48 \qquad \textbf{(B) } 72 \qquad \textbf{(C) } 96 \qquad \textbf{(D) } 144 \qquad \textbf{(E) } 192</math>
Line 25: Line 36:
 
</asy>
 
</asy>
  
We know that <math>\triangle AUV \sim \triangle AMC</math>, and since the ratios of its sides are <math>\frac{1}{2}</math>, the ratio of of their areas is <math>(\frac{1}{2})^2=\frac{1}{4}</math>.  
+
We know that <math>\triangle AUV \sim \triangle AMC</math>, and since the ratios of its sides are <math>\frac{1}{2}</math>, the ratio of of their areas is <math>\left(\frac{1}{2}\right)^2=\frac{1}{4}</math>.  
  
 
If <math>\triangle AUV</math> is <math>\frac{1}{4}</math> the area of <math>\triangle AMC</math>, then trapezoid <math>MUVC</math> is <math>\frac{3}{4}</math> the area of <math>\triangle AMC</math>.  
 
If <math>\triangle AUV</math> is <math>\frac{1}{4}</math> the area of <math>\triangle AMC</math>, then trapezoid <math>MUVC</math> is <math>\frac{3}{4}</math> the area of <math>\triangle AMC</math>.  
Line 56: Line 67:
 
</asy>
 
</asy>
  
Since we know that all medians of a triangle intersect at the incenter, we know that <math>\overline{AB}</math> passes through point <math>P</math>. We also know that medians of a triangle divide each other into segments of ratio <math>2:1</math>. Knowing this, we can see that <math>\overline{PC}:\overline{UP}=2:1</math>, and since the two segments sum to <math>12</math>, <math>\overline{PC}</math> and <math>\overline{UP}</math> are <math>8</math> and <math>4</math>, respectively.
+
Since we know that all medians of a triangle intersect at the centroid, we know that <math>\overline{AB}</math> passes through point <math>P</math>. We also know that medians of a triangle divide each other into segments of ratio <math>2:1</math>. Knowing this, we can see that <math>\overline{PC}:\overline{UP}=2:1</math>, and since the two segments sum to <math>12</math>, <math>\overline{PC}</math> and <math>\overline{UP}</math> are <math>8</math> and <math>4</math>, respectively.
  
 
Finally knowing that the medians divide the triangle into <math>6</math> sections of equal area, finding the area of <math>\triangle PUM</math> is enough. <math>\overline{PC} = \overline{MP} = 8</math>.
 
Finally knowing that the medians divide the triangle into <math>6</math> sections of equal area, finding the area of <math>\triangle PUM</math> is enough. <math>\overline{PC} = \overline{MP} = 8</math>.
  
The area of <math>\triangle PUM = \frac{4\cdot8}{2}=16</math>. Multiplying this by <math>6</math> gives us <math>6\cdot16=\boxed{\textbf{(C) }96}</math> ~quacker88
+
The area of <math>\triangle PUM = \frac{4\cdot8}{2}=16</math>. Multiplying this by <math>6</math> gives us <math>6\cdot16=\boxed{\textbf{(C) }96}</math>  
  
==Video Solution==
+
~quacker88
 +
 
 +
==Solution 4 (Triangles)==
 +
<asy>
 +
draw((-4,0)--(4,0)--(0,12)--cycle);
 +
draw((-2,6)--(4,0));
 +
draw((2,6)--(-4,0));
 +
draw((-2,6)--(2,6));
 +
label("M", (-4,0), W);
 +
label("C", (4,0), E);
 +
label("A", (0, 12), N);
 +
label("V", (2, 6), NE);
 +
label("U", (-2, 6), NW);
 +
label("P", (0, 3.6), S);
 +
</asy>
 +
We know that <math>AU = UM</math>, <math>AV = VC</math>, so <math>UV = \frac{1}{2} MC</math>.
 +
 
 +
As <math>\angle UPM = \angle VPC = 90</math>, we can see that <math>\triangle UPM \cong \triangle VPC</math> and <math>\triangle UVP \sim \triangle MPC</math> with a side ratio of <math>1 : 2</math>.
 +
 
 +
So <math>UP = VP = 4</math>, <math>MP = PC = 8</math>.
 +
 
 +
With that, we can see that <math>[\triangle UPM] = 16</math>, and the area of trapezoid <math>MUVC</math> is 72.
 +
 
 +
As said in solution 1, <math>[\triangle AMC] = 72  /  \frac{3}{4} = \boxed{\textbf{(C) } 96}</math>.
 +
 
 +
-QuadraticFunctions, solution 1 by ???
 +
 
 +
==Solution 5 (Only Pythagorean Theorem)==
 +
<asy>
 +
draw((-4,0)--(4,0)--(0,12)--cycle);
 +
draw((-2,6)--(4,0));
 +
draw((2,6)--(-4,0));
 +
draw((0,12)--(0,0));
 +
label("M", (-4,0), W);
 +
label("C", (4,0), E);
 +
label("A", (0, 12), N);
 +
label("V", (2, 6), NE);
 +
label("U", (-2, 6), NW);
 +
label("P", (0.5, 4), E);
 +
label("B", (0, 0), S);
 +
 
 +
</asy>
 +
 
 +
Let <math>AB</math> be the height. Since medians divide each other into a <math>2:1</math> ratio, and the medians have length 12, we have <math>PC=MP=8</math> and <math>UP=VP=4</math>. From right triangle <math>\triangle{MUP}</math>, <cmath>MU^2=MP^2+UP^2=8^2+4^2=80,</cmath> so <math>MU=\sqrt{80}=4\sqrt{5}</math>. Since <math>CU</math> is a median, <math>AM=8\sqrt{5}</math>. From right triangle <math>\triangle{MPC}</math>, <cmath>MC^2=MP^2+PC^2=8^2+8^2=128,</cmath> which implies <math>MC=\sqrt{128}=8\sqrt{2}</math>. By symmetry <math>MB=\dfrac{8\sqrt{2}}{2}=4\sqrt{2}</math>.
 +
 
 +
Applying the Pythagorean Theorem to right triangle <math>\triangle{MAB}</math> gives <math>AB^2=AM^2-MB^2=8\sqrt{5}^2-4\sqrt{2}^2=288</math>, so <math>AB=\sqrt{288}=12\sqrt{2}</math>. Then the area of <math>\triangle{AMC}</math> is <cmath>\dfrac{AB \cdot MC}{2}=\dfrac{8\sqrt{2} \cdot 12\sqrt{2}}{2}=\dfrac{96 \cdot 2}{2}=\boxed{\textbf{(C) }96}</cmath>
 +
 
 +
==Solution 6 (Drawing)==
 +
By similarity, the area of <math>AUV</math> is equal to <math>\frac{1}{4}</math>.
 +
 
 +
The area of <math>UVCM</math> is equal to 72.
 +
 
 +
Assuming the total area of the triangle is S, the equation will be : <math>\frac{3}{4}</math>S = 72.
 +
 
 +
S = <math>\boxed{\textbf{(C) }96}</math>
 +
 
 +
==Solution 7==
 +
Given a triangle with perpendicular medians with lengths <math>x</math> and <math>y</math>, the area will be <math>\frac{2xy}{3}=\boxed{\textbf{(C) }96}</math>.
 +
 
 +
==Solution 8 (Fastest)==
 +
Connect the line segment <math>UV</math> and it's easy to see quadrilateral <math>UVMC</math> has an area of the product of its diagonals divided by <math>2</math> which is <math>72</math>. Now, solving for triangle <math>AUV</math> could be an option, but the drawing shows the area of <math>AUV</math> will be less than the quadrilateral meaning the the area of <math>AMC</math> is less than <math>72*2</math> but greater than <math>72</math>, leaving only one possible answer choice, <math>\boxed{\textbf{(C) } 96}</math>.
 +
 
 +
-Rohan S.
 +
 
 +
==Solution 9==
 +
<asy>
 +
draw((-4,0)--(4,0)--(0,12)--cycle);
 +
draw((-2,6)--(4,0));
 +
draw((2,6)--(-4,0));
 +
draw((0,12)--(0,0));
 +
label("M", (-4,0), W);
 +
label("C", (4,0), E);
 +
label("A", (0, 12), N);
 +
label("V", (2, 6), NE);
 +
label("U", (-2, 6), NW);
 +
label("P", (0.5, 4), E);
 +
label("B", (0, 0), S);
 +
</asy>
 +
 
 +
Connect <math>AP</math>, and let <math>B</math> be the point where <math>AP</math> intersects <math>MC</math>. <math>MB=CB</math> because all medians of a triangle intersect at one point, which in this case is <math>P</math>. <math>MP:PV=2:1</math> because the point at which all medians intersect divides the medians into segments of ratio <math>2:1</math>, so <math>MP=8</math> and similarly <math>CP=8</math>. We apply the Pythagorean Theorem to triangle <math>MPC</math> and get <math>MC=\sqrt{128}=8\sqrt{2}</math>. The area of triangle <math>MPC</math> is <math>\dfrac{MP\cdot CP}{2}=32</math>, and that must equal to <math>\dfrac{MC\cdot BP}{2}</math>, so <math>BP=\dfrac{8}{\sqrt{2}}=4\cdot\sqrt{2}</math>. <math>BP=\dfrac{1}{3}BA</math>, so <math>BA=12\sqrt{2}</math>. The area of triangle <math>AMC</math> is equal to <math>\dfrac{MC\cdot BA}{2}=\dfrac{8 \cdot \sqrt{2} \cdot 12 \cdot \sqrt{2}}{2}=\boxed{\textbf{(C)}\ 96}</math>.
 +
 
 +
-SmileKat32
 +
 
 +
== Solution 10 (High IQ) ==
 +
<math>[\square MUVC] = 72</math>. Let intersection of line <math>AP</math> and base <math>MC</math> be <math>B</math> <cmath> [AUV]=[MUB]=[UVB]=[BVC] \implies \left[\frac{\triangle AMC}{4}\right] = \left[\frac{\square MUVC}{3}\right] \implies [AMC] = \boxed{\textbf{(C)}\ 96}</cmath>
 +
 
 +
~herobrine-india 
 +
 
 +
==Solution 11 (Kite)==
 +
<asy>
 +
draw((-4,0)--(4,0)--(0,12)--cycle);
 +
draw((-2,6)--(4,0));
 +
draw((2,6)--(-4,0));
 +
draw((-2,6)--(2,6));
 +
label("M", (-4,0), W);
 +
label("C", (4,0), E);
 +
label("A", (0, 12), N);
 +
label("V", (2, 6), NE);
 +
label("U", (-2, 6), NW);
 +
label("P", (0, 3.6), S);
 +
</asy>
 +
 
 +
Since <math>\overline{MV}</math> and <math>\overline{CU}</math> intersect at a right angle, this means <math>MUVC</math> is a kite. Hence, the area of this kite is <math>\frac{12 \cdot 12}{2} = 72</math>.
 +
 
 +
Also, notice that <math>\triangle AUV \sim \triangle AMC</math> by AAA Similarity. Since the ratios of its sides is <math>\frac{1}{2}</math>, the ratios of the area is <math>\left(\frac{1}{2}\right)^2=\frac{1}{4}</math>. Therefore,
 +
 
 +
<cmath>[AMC] = [MUVC] + \frac{1}{4} \cdot [AMC]</cmath>
 +
 
 +
Simplifying gives us <math>\frac{3}{4} \cdot [AMC] = 72</math>, so <math>[AMC] = 72 \cdot \frac{4}{3} = \boxed{\textbf{(C)}\ 96}</math>
 +
 
 +
~MrThinker
 +
 
 +
==Solution 12 (Educated guessing)==
 +
Horizontally translate line <math>\overline{UC}</math> until point <math>U</math> is at point <math>V</math>, with <math>C</math> subsequently at <math>C'</math>, and then connect up <math>C'</math> and <math>C</math> to create <math>\triangle MVC'</math>, which is a right triangle.
 +
 
 +
<asy>
 +
draw((-4,0)--(4,0)--(0,12)--cycle);
 +
draw((-2,6)--(4,0));
 +
draw((2,6)--(-4,0));
 +
draw((-2,6)--(2,6));
 +
draw((2,6)--(8,0));
 +
draw((4,0)--(8,0));
 +
label("M", (-4,0), W);
 +
label("C", (4,0), S);
 +
label("A", (0, 12), N);
 +
label("V", (2, 6), NE);
 +
label("U", (-2, 6), NW);
 +
label("P", (0, 3.6), S);
 +
label("C'", (8,0), E);
 +
</asy>
 +
 
 +
Notice that <math>\triangle MVC'</math> = <math>12 \cdot 12 \cdot \frac{1}{2} = 72</math>, and <math>\triangle MVC'</math> = <math>\triangle MVC + \triangle MUV</math> (since the latter has the same base and height as the sub-triangle <math>\triangle CVC'</math> inside <math>\triangle MVC'</math>).
 +
 
 +
From this, we can deduce that <math>\textbf{(B)}</math> cannot be true, since an incomplete part of <math>\triangle AMC</math> is equal to it. We can also deduce that <math>\textbf{(D)}</math> also cannot be true, since the unknown triangle <math>\triangle AUV = \triangle MUV</math>, and <math>\triangle MUV = \triangle CVC' < \triangle MVC'</math>. Therefore, the answer must be between <math>72</math> and <math>144</math>, leaving <math>\boxed{\textbf{(C)}\ 96}</math> as the only possible correct answer.
 +
 
 +
~DifrractSliver
 +
 
 +
==Video Solution 1==
 +
 
 +
Education, The Study of Everything
 +
 
 +
https://youtu.be/0TslJ3aDXac
 +
 
 +
==Video Solution 2==
 
https://youtu.be/ZGwAasE32Y4
 
https://youtu.be/ZGwAasE32Y4
  
 
~IceMatrix
 
~IceMatrix
 +
 +
==Video Solution 3==
 +
https://youtu.be/7ZvKOYuwSnE
 +
 +
~savannahsolver
 +
 +
== Video Solution 4 by OmegaLearn ==
 +
https://youtu.be/4_x1sgcQCp4?t=2067
 +
 +
~ pi_is_3.14
  
 
==See Also==
 
==See Also==

Revision as of 17:05, 12 November 2023

Problem 12

Triangle $AMC$ is isosceles with $AM = AC$. Medians $\overline{MV}$ and $\overline{CU}$ are perpendicular to each other, and $MV=CU=12$. What is the area of $\triangle AMC?$

[asy] draw((-4,0)--(4,0)--(0,12)--cycle); draw((-2,6)--(4,0)); draw((2,6)--(-4,0)); label("M", (-4,0), W); label("C", (4,0), E); label("A", (0, 12), N); label("V", (2, 6), NE); label("U", (-2, 6), NW); label("P", (0, 3.6), S); [/asy]

$\textbf{(A) } 48 \qquad \textbf{(B) } 72 \qquad \textbf{(C) } 96 \qquad \textbf{(D) } 144 \qquad \textbf{(E) } 192$

Solution 1

Since quadrilateral $UVCM$ has perpendicular diagonals, its area can be found as half of the product of the length of the diagonals. Also note that $\triangle AUV$ has $\frac 14$ the area of triangle $AMC$ by similarity, so $[UVCM]=\frac 34\cdot [AMC].$ Thus, \[\frac 12 \cdot 12\cdot 12=\frac 34 \cdot [AMC]\] \[72=\frac 34\cdot [AMC]\] \[[AMC]=96\rightarrow \boxed{\textbf{(C)}}.\]

Solution 2 (Trapezoid)

[asy] draw((-4,0)--(4,0)--(0,12)--cycle); draw((-2,6)--(4,0)); draw((2,6)--(-4,0)); draw((-2,6)--(2,6)); label("M", (-4,0), W); label("C", (4,0), E); label("A", (0, 12), N); label("V", (2, 6), NE); label("U", (-2, 6), NW); label("P", (0, 3.6), S); [/asy]

We know that $\triangle AUV \sim \triangle AMC$, and since the ratios of its sides are $\frac{1}{2}$, the ratio of of their areas is $\left(\frac{1}{2}\right)^2=\frac{1}{4}$.

If $\triangle AUV$ is $\frac{1}{4}$ the area of $\triangle AMC$, then trapezoid $MUVC$ is $\frac{3}{4}$ the area of $\triangle AMC$.

Let's call the intersection of $\overline{UC}$ and $\overline{MV}$ $P$. Let $\overline{UP}=x$. Then $\overline{PC}=12-x$. Since $\overline{UC}  \perp \overline{MV}$, $\overline{UP}$ and $\overline{CP}$ are heights of triangles $\triangle MUV$ and $\triangle MCV$, respectively. Both of these triangles have base $12$.

Area of $\triangle MUV = \frac{x\cdot12}{2}=6x$

Area of $\triangle MCV = \frac{(12-x)\cdot12}{2}=72-6x$

Adding these two gives us the area of trapezoid $MUVC$, which is $6x+(72-6x)=72$.

This is $\frac{3}{4}$ of the triangle, so the area of the triangle is $\frac{4}{3}\cdot{72}=\boxed{\textbf{(C) } 96}$ ~quacker88, diagram by programjames1

Solution 3 (Medians)

Draw median $\overline{AB}$. [asy] draw((-4,0)--(4,0)--(0,12)--cycle); draw((-2,6)--(4,0)); draw((2,6)--(-4,0)); draw((0,12)--(0,0)); label("M", (-4,0), W); label("C", (4,0), E); label("A", (0, 12), N); label("V", (2, 6), NE); label("U", (-2, 6), NW); label("P", (0.5, 4), E); label("B", (0, 0), S); [/asy]

Since we know that all medians of a triangle intersect at the centroid, we know that $\overline{AB}$ passes through point $P$. We also know that medians of a triangle divide each other into segments of ratio $2:1$. Knowing this, we can see that $\overline{PC}:\overline{UP}=2:1$, and since the two segments sum to $12$, $\overline{PC}$ and $\overline{UP}$ are $8$ and $4$, respectively.

Finally knowing that the medians divide the triangle into $6$ sections of equal area, finding the area of $\triangle PUM$ is enough. $\overline{PC} = \overline{MP} = 8$.

The area of $\triangle PUM = \frac{4\cdot8}{2}=16$. Multiplying this by $6$ gives us $6\cdot16=\boxed{\textbf{(C) }96}$

~quacker88

Solution 4 (Triangles)

[asy] draw((-4,0)--(4,0)--(0,12)--cycle); draw((-2,6)--(4,0)); draw((2,6)--(-4,0)); draw((-2,6)--(2,6)); label("M", (-4,0), W); label("C", (4,0), E); label("A", (0, 12), N); label("V", (2, 6), NE); label("U", (-2, 6), NW); label("P", (0, 3.6), S); [/asy] We know that $AU = UM$, $AV = VC$, so $UV = \frac{1}{2} MC$.

As $\angle UPM = \angle VPC = 90$, we can see that $\triangle UPM \cong \triangle VPC$ and $\triangle UVP \sim \triangle MPC$ with a side ratio of $1 : 2$.

So $UP = VP = 4$, $MP = PC = 8$.

With that, we can see that $[\triangle UPM] = 16$, and the area of trapezoid $MUVC$ is 72.

As said in solution 1, $[\triangle AMC] = 72  /  \frac{3}{4} = \boxed{\textbf{(C) } 96}$.

-QuadraticFunctions, solution 1 by ???

Solution 5 (Only Pythagorean Theorem)

[asy] draw((-4,0)--(4,0)--(0,12)--cycle); draw((-2,6)--(4,0)); draw((2,6)--(-4,0)); draw((0,12)--(0,0)); label("M", (-4,0), W); label("C", (4,0), E); label("A", (0, 12), N); label("V", (2, 6), NE); label("U", (-2, 6), NW); label("P", (0.5, 4), E); label("B", (0, 0), S);  [/asy]

Let $AB$ be the height. Since medians divide each other into a $2:1$ ratio, and the medians have length 12, we have $PC=MP=8$ and $UP=VP=4$. From right triangle $\triangle{MUP}$, \[MU^2=MP^2+UP^2=8^2+4^2=80,\] so $MU=\sqrt{80}=4\sqrt{5}$. Since $CU$ is a median, $AM=8\sqrt{5}$. From right triangle $\triangle{MPC}$, \[MC^2=MP^2+PC^2=8^2+8^2=128,\] which implies $MC=\sqrt{128}=8\sqrt{2}$. By symmetry $MB=\dfrac{8\sqrt{2}}{2}=4\sqrt{2}$.

Applying the Pythagorean Theorem to right triangle $\triangle{MAB}$ gives $AB^2=AM^2-MB^2=8\sqrt{5}^2-4\sqrt{2}^2=288$, so $AB=\sqrt{288}=12\sqrt{2}$. Then the area of $\triangle{AMC}$ is \[\dfrac{AB \cdot MC}{2}=\dfrac{8\sqrt{2} \cdot 12\sqrt{2}}{2}=\dfrac{96 \cdot 2}{2}=\boxed{\textbf{(C) }96}\]

Solution 6 (Drawing)

By similarity, the area of $AUV$ is equal to $\frac{1}{4}$.

The area of $UVCM$ is equal to 72.

Assuming the total area of the triangle is S, the equation will be : $\frac{3}{4}$S = 72.

S = $\boxed{\textbf{(C) }96}$

Solution 7

Given a triangle with perpendicular medians with lengths $x$ and $y$, the area will be $\frac{2xy}{3}=\boxed{\textbf{(C) }96}$.

Solution 8 (Fastest)

Connect the line segment $UV$ and it's easy to see quadrilateral $UVMC$ has an area of the product of its diagonals divided by $2$ which is $72$. Now, solving for triangle $AUV$ could be an option, but the drawing shows the area of $AUV$ will be less than the quadrilateral meaning the the area of $AMC$ is less than $72*2$ but greater than $72$, leaving only one possible answer choice, $\boxed{\textbf{(C) } 96}$.

-Rohan S.

Solution 9

[asy] draw((-4,0)--(4,0)--(0,12)--cycle); draw((-2,6)--(4,0)); draw((2,6)--(-4,0)); draw((0,12)--(0,0)); label("M", (-4,0), W); label("C", (4,0), E); label("A", (0, 12), N); label("V", (2, 6), NE); label("U", (-2, 6), NW); label("P", (0.5, 4), E); label("B", (0, 0), S); [/asy]

Connect $AP$, and let $B$ be the point where $AP$ intersects $MC$. $MB=CB$ because all medians of a triangle intersect at one point, which in this case is $P$. $MP:PV=2:1$ because the point at which all medians intersect divides the medians into segments of ratio $2:1$, so $MP=8$ and similarly $CP=8$. We apply the Pythagorean Theorem to triangle $MPC$ and get $MC=\sqrt{128}=8\sqrt{2}$. The area of triangle $MPC$ is $\dfrac{MP\cdot CP}{2}=32$, and that must equal to $\dfrac{MC\cdot BP}{2}$, so $BP=\dfrac{8}{\sqrt{2}}=4\cdot\sqrt{2}$. $BP=\dfrac{1}{3}BA$, so $BA=12\sqrt{2}$. The area of triangle $AMC$ is equal to $\dfrac{MC\cdot BA}{2}=\dfrac{8 \cdot \sqrt{2} \cdot 12 \cdot \sqrt{2}}{2}=\boxed{\textbf{(C)}\ 96}$.

-SmileKat32

Solution 10 (High IQ)

$[\square MUVC] = 72$. Let intersection of line $AP$ and base $MC$ be $B$ \[[AUV]=[MUB]=[UVB]=[BVC] \implies \left[\frac{\triangle AMC}{4}\right] = \left[\frac{\square MUVC}{3}\right] \implies [AMC] = \boxed{\textbf{(C)}\ 96}\]

~herobrine-india

Solution 11 (Kite)

[asy] draw((-4,0)--(4,0)--(0,12)--cycle); draw((-2,6)--(4,0)); draw((2,6)--(-4,0)); draw((-2,6)--(2,6)); label("M", (-4,0), W); label("C", (4,0), E); label("A", (0, 12), N); label("V", (2, 6), NE); label("U", (-2, 6), NW); label("P", (0, 3.6), S); [/asy]

Since $\overline{MV}$ and $\overline{CU}$ intersect at a right angle, this means $MUVC$ is a kite. Hence, the area of this kite is $\frac{12 \cdot 12}{2} = 72$.

Also, notice that $\triangle AUV \sim \triangle AMC$ by AAA Similarity. Since the ratios of its sides is $\frac{1}{2}$, the ratios of the area is $\left(\frac{1}{2}\right)^2=\frac{1}{4}$. Therefore,

\[[AMC] = [MUVC] + \frac{1}{4} \cdot [AMC]\]

Simplifying gives us $\frac{3}{4} \cdot [AMC] = 72$, so $[AMC] = 72 \cdot \frac{4}{3} = \boxed{\textbf{(C)}\ 96}$

~MrThinker

Solution 12 (Educated guessing)

Horizontally translate line $\overline{UC}$ until point $U$ is at point $V$, with $C$ subsequently at $C'$, and then connect up $C'$ and $C$ to create $\triangle MVC'$, which is a right triangle.

[asy] draw((-4,0)--(4,0)--(0,12)--cycle); draw((-2,6)--(4,0)); draw((2,6)--(-4,0)); draw((-2,6)--(2,6)); draw((2,6)--(8,0)); draw((4,0)--(8,0)); label("M", (-4,0), W); label("C", (4,0), S); label("A", (0, 12), N); label("V", (2, 6), NE); label("U", (-2, 6), NW); label("P", (0, 3.6), S); label("C'", (8,0), E); [/asy]

Notice that $\triangle MVC'$ = $12 \cdot 12 \cdot \frac{1}{2} = 72$, and $\triangle MVC'$ = $\triangle MVC + \triangle MUV$ (since the latter has the same base and height as the sub-triangle $\triangle CVC'$ inside $\triangle MVC'$).

From this, we can deduce that $\textbf{(B)}$ cannot be true, since an incomplete part of $\triangle AMC$ is equal to it. We can also deduce that $\textbf{(D)}$ also cannot be true, since the unknown triangle $\triangle AUV = \triangle MUV$, and $\triangle MUV = \triangle CVC' < \triangle MVC'$. Therefore, the answer must be between $72$ and $144$, leaving $\boxed{\textbf{(C)}\ 96}$ as the only possible correct answer.

~DifrractSliver

Video Solution 1

Education, The Study of Everything

https://youtu.be/0TslJ3aDXac

Video Solution 2

https://youtu.be/ZGwAasE32Y4

~IceMatrix

Video Solution 3

https://youtu.be/7ZvKOYuwSnE

~savannahsolver

Video Solution 4 by OmegaLearn

https://youtu.be/4_x1sgcQCp4?t=2067

~ pi_is_3.14

See Also

2020 AMC 10A (ProblemsAnswer KeyResources)
Preceded by
Problem 11
Followed by
Problem 13
1 2 3 4 5 6 7 8 9 10 11 12 13 14 15 16 17 18 19 20 21 22 23 24 25
All AMC 10 Problems and Solutions

The problems on this page are copyrighted by the Mathematical Association of America's American Mathematics Competitions. AMC logo.png